Question 2 of 25
Which of the following is an equation of a line parallel to the equation
y = 4x + 1?
O A. y=1x-2
O B. y=-x-2
O C. y=-4x-2
O D. y = 4x - 2
DOSUBMIT

Answers

Answer 1

Answer:

y - 1 = 4x - 20.

The slope of the line y = 4x +1 is the coefficient of x, so the slope is 4. Parallel lines have the same slope, so the slope of the "other" line is also 4. Using the point-slope form of a line, the equation of the line in question is: y - 1 = 4(x - 5). Distributing 4, we get y - 1 = 4x - 20. i dont know correct me if im wrong


Related Questions

Doyle Company issued $500,000 of 10-year, 7 percent bonds on January 1, 2018. The bonds were issued at face value. Interest is payable in cash on December 31 of each year. Doyle immediately invested the proceeds from the bond issue in land. The land was leased for an annual $125,000 of cash revenue, which was collected on December 31 of each year, beginning December 31, 2018

Answers

Answer:

f

Step-by-step explanation:

Please help!!

Find BD​

Answers

Answer:  [tex]8\sqrt{2}[/tex]

==========================================================

Work Shown:

Focus entirely on triangle ABD (or on triangle BCD; both are identical)

The two legs of this triangle are AB = 8 and AD = 8. The hypotenuse is unknown, so we'll say BD = x.

Apply the pythagorean theorem.

[tex]a^2 + b^2 = c^2\\\\c = \sqrt{a^2 + b^2}\\\\x = \sqrt{8^2 + 8^2}\\\\x = \sqrt{2*8^2}\\\\x = \sqrt{8^2*2}\\\\x = \sqrt{8^2}*\sqrt{2}\\\\x = 8\sqrt{2}\\\\[/tex]

So that's why the diagonal BD is exactly [tex]8\sqrt{2}\\\\[/tex] units long

Side note: [tex]8\sqrt{2} \approx 11.3137[/tex]

The distance between Ali's house and 1 point
college is exactly 135 miles. If she
drove 2/3 of the distance in 135
minutes. What was her average speed
in miles per hour?

Answers

First we have to figure out how long it would take for the full voyage and that would be 135 + (135 x 1/3) and the answer to that would be 135 + 45 = 180 and that means that 180 is the total minutes it would take to travel the whole trip, now we have to calculate average speed which would be 135(distance)/180(time) which would end up being 135 miles/ 3 hours, now we divide the entire equation by 3 which would be 45/1
CONCLUSION ——————————
Ali would be driving 45 miles per hour

Ali's average speed was 40 miles per hour.

What is an average speed?

The total distance traveled is to be divided by the total time consumed brings us the average speed.

How to calculate the average speed of Ali?

The total distance between the college from Ali's house is 135 miles.

She drove 2/3rd of the total distance in 135 minutes.

She drove =135*2/3miles

=90miles.

Ali can drive 90miles in 135 mins.

Therefore, her average speed is: 90*60/135 miles per hour.

=40 miles per hour.

Learn about average speed here :

https://brainly.in/question/14787217

#SPJ2

190 of 7
6 7 8 9 10
-3
4
5
6
The slope of the line shown in the graph is
and the intercept of the line is

Answers

Answer:slope 2/3

Y-int 6

Step-by-step explanation:

A rectangle has a length of 7 in. and a width of 2 in. if the rectangle is enlarged using a scale factor of 1.5, what will be the perimeter of the new rectangle

Answers

Answer:

27 inch

Step-by-step explanation:

Current perimeter=18

New perimeter=18*1.5=27 in

the number of multiples of a given number is infinite ( )​

Answers

Answer:

make an 8 horizontal

oooookkkk

Answer:

TRUE

The number of multiples of a given number is finite is a false statement. The number of multiples of a given number is infinite.

Examples:

Multiples of 2 = 2,4,6,8,10,…..

Multiples of 3 = 3,6,9,12,15,18,…

Multiples of 4 = 4, 8, 12, 16, 120, 24….

∴ The number of multiples of a given number is infinite .

Answer From Gauth Math

5x-22 3x +105 x minus 22 3 X + 10 ​

Answers

-291x+10

:)))))) Have fun

A shop sells a particular of video recorder. Assuming that the weekly demand for the video recorder is a Poisson variable with the mean 3, find the probability that the shop sells. . (a) At least 3 in a week. (b) At most 7 in a week. (c) More than 20 in a month (4 weeks).

Answers

Answer:

a) 0.5768 = 57.68% probability that the shop sells at least 3 in a week.

b) 0.988 = 98.8% probability that the shop sells at most 7 in a week.

c) 0.0104 = 1.04% probability that the shop sells more than 20 in a month.

Step-by-step explanation:

For questions a and b, the Poisson distribution is used, while for question c, the normal approximation is used.

Poisson distribution:

In a Poisson distribution, the probability that X represents the number of successes of a random variable is given by the following formula:

[tex]P(X = x) = \frac{e^{-\lambda}*\lambda^{x}}{(x)!}[/tex]

In which

x is the number of successes

e = 2.71828 is the Euler number

[tex]\lambda[/tex] is the mean in the given interval.

Normal Probability Distribution

Problems of normal distributions can be solved using the z-score formula.

In a set with mean [tex]\mu[/tex] and standard deviation [tex]\sigma[/tex], the z-score of a measure X is given by:

[tex]Z = \frac{X - \mu}{\sigma}[/tex]

The Z-score measures how many standard deviations the measure is from the mean. After finding the Z-score, we look at the z-score table and find the p-value associated with this z-score. This p-value is the probability that the value of the measure is smaller than X, that is, the percentile of X. Subtracting 1 by the p-value, we get the probability that the value of the measure is greater than X.

The Poisson distribution can be approximated to the normal with [tex]\mu = \lambda, \sigma = \sqrt{\lambda}[/tex], if [tex]\lambda>10[/tex].

Poisson variable with the mean 3

This means that [tex]\lambda= 3[/tex].

(a) At least 3 in a week.

This is [tex]P(X \geq 3)[/tex]. So

[tex]P(X \geq 3) = 1 - P(X < 3)[/tex]

In which:

[tex]P(X < 3) = P(X = 0) + P(X = 1) + P(X = 2)[/tex]

Then

[tex]P(X = x) = \frac{e^{-\lambda}*\lambda^{x}}{(x)!}[/tex]

[tex]P(X = 0) = \frac{e^{-3}*3^{0}}{(0)!} = 0.0498[/tex]

[tex]P(X = 1) = \frac{e^{-3}*3^{1}}{(1)!} = 0.1494[/tex]

[tex]P(X = 2) = \frac{e^{-3}*3^{2}}{(2)!} = 0.2240[/tex]

So

[tex]P(X < 3) = P(X = 0) + P(X = 1) + P(X = 2) = 0.0498 + 0.1494 + 0.2240 = 0.4232[/tex]

[tex]P(X < 3) = P(X = 0) + P(X = 1) + P(X = 2) = 1 - 0.4232 = 0.5768[/tex]

0.5768 = 57.68% probability that the shop sells at least 3 in a week.

(b) At most 7 in a week.

This is:

[tex]P(X \leq 7) = P(X = 0) + P(X = 1) + P(X = 2) + P(X = 3) + P(X = 4) + P(X = 5) + P(X = 6) + P(X = 7)[/tex]

In which

[tex]P(X = x) = \frac{e^{-\lambda}*\lambda^{x}}{(x)!}[/tex]

[tex]P(X = 0) = \frac{e^{-3}*3^{0}}{(0)!} = 0.0498[/tex]

[tex]P(X = 1) = \frac{e^{-3}*3^{1}}{(1)!} = 0.1494[/tex]

[tex]P(X = 2) = \frac{e^{-3}*3^{2}}{(2)!} = 0.2240[/tex]

[tex]P(X = 3) = \frac{e^{-3}*3^{3}}{(3)!} = 0.2240[/tex]

[tex]P(X = 4) = \frac{e^{-3}*3^{4}}{(4)!} = 0.1680[/tex]

[tex]P(X = 5) = \frac{e^{-3}*3^{5}}{(5)!} = 0.1008[/tex]

[tex]P(X = 6) = \frac{e^{-3}*3^{6}}{(6)!} = 0.0504[/tex]

[tex]P(X = 7) = \frac{e^{-3}*3^{7}}{(7)!} = 0.0216[/tex]

Then

[tex]P(X \leq 7) = P(X = 0) + P(X = 1) + P(X = 2) + P(X = 3) + P(X = 4) + P(X = 5) + P(X = 6) + P(X = 7) = 0.0498 + 0.1494 + 0.2240 + 0.2240 + 0.1680 + 0.1008 + 0.0504 + 0.0216 = 0.988[/tex]

0.988 = 98.8% probability that the shop sells at most 7 in a week.

(c) More than 20 in a month (4 weeks).

4 weeks, so:

[tex]\mu = \lambda = 4(3) = 12[/tex]

[tex]\sigma = \sqrt{\lambda} = \sqrt{12}[/tex]

The probability, using continuity correction, is P(X > 20 + 0.5) = P(X > 20.5), which is 1 subtracted by the p-value of Z when X = 20.5.

[tex]Z = \frac{X - \mu}{\sigma}[/tex]

[tex]Z = \frac{20 - 12}{\sqrt{12}}[/tex]

[tex]Z = 2.31[/tex]

[tex]Z = 2.31[/tex] has a p-value of 0.9896.

1 - 0.9896 = 0.0104

0.0104 = 1.04% probability that the shop sells more than 20 in a month.

The probability of the selling the video recorders for considered cases are:

P(At least 3 in a week) = 0.5768 approximately.P(At most 7 in a week) = 0.9881 approximately.P( more than 20 in a month) = 0.0839 approximately.

What are some of the properties of Poisson distribution?

Let X ~ Pois(λ)

Then we have:

E(X) = λ = Var(X)

Since standard deviation is square root (positive) of variance,

Thus,

Standard deviation of X = [tex]\sqrt{\lambda}[/tex]

Its probability function is given by

f(k; λ) = Pr(X = k) = [tex]\dfrac{\lambda^{k}e^{-\lambda}}{k!}[/tex]

For this case, let we have:

X = the number of weekly demand of video recorder for the considered shop.

Then, by the given data, we have:

X ~ Pois(λ=3)


Evaluating each event's probability:

Case 1: At least 3 in a week.

[tex]P(X > 3) = 1- P(X \leq 2) = \sum_{i=0}^{2}P(X=i) = \sum_{i=0}^{2} \dfrac{3^ie^{-3}}{i!}\\\\P(X > 3) = 1 - e^{-3} \times \left( 1 + 3 + 9/2\right) \approx 1 - 0.4232 = 0.5768[/tex]

Case 2: At most 7 in a week.

[tex]P(X \leq 7) = \sum_{i=0}^{7}P(X=i) = \sum_{i=0}^{7} \dfrac{3^ie^{-3}}{i!}\\\\P(X \leq 7) = e^{-3} \times \left( 1 + 3 + 9/2 + 27/6 + 81/24 + 243/120 + 729/720 + 2187/5040\right)\\\\P(X \leq 7) \approx 0.9881[/tex]

Case 3: More than 20 in a month(4 weeks)

That means more than 5 in a week on average.

[tex]P(X > 5) = 1- P(X \leq 5) =\sum_{i=0}^{5}P(X=i) = \sum_{i=0}^{5} \dfrac{3^ie^{-3}}{i!}\\\\P(X > 5) = 1- e^{-3}( 1 + 3 + 9/2 + 27/6 + 81/24 + 243/120)\\\\P(X > 5) \approx 1 - 0.9161 \\ P(X > 5) \approx 0.0839[/tex]


Thus, the probability of the selling the video recorders for considered cases are:

P(At least 3 in a week) = 0.5768 approximately.P(At most 7 in a week) = 0.9881 approximately.P( more than 20 in a month) = 0.0839 approximately.

Learn more about poisson distribution here:

https://brainly.com/question/7879375

If a normally distributed population has a mean (mu) that equals 100 with a standard deviation (sigma) of 18, what will be the computed z-score with a sample mean (x-bar) of 106 from a sample size of 9?

Answers

Answer:

Z = 1

Step-by-step explanation:

To solve this question, we need to understand the normal probability distribution and the central limit theorem.

Normal Probability Distribution

Problems of normal distributions can be solved using the z-score formula.

In a set with mean [tex]\mu[/tex] and standard deviation [tex]\sigma[/tex], the z-score of a measure X is given by:

[tex]Z = \frac{X - \mu}{\sigma}[/tex]

The Z-score measures how many standard deviations the measure is from the mean. After finding the Z-score, we look at the z-score table and find the p-value associated with this z-score. This p-value is the probability that the value of the measure is smaller than X, that is, the percentile of X. Subtracting 1 by the p-value, we get the probability that the value of the measure is greater than X.

Central Limit Theorem

The Central Limit Theorem establishes that, for a normally distributed random variable X, with mean [tex]\mu[/tex] and standard deviation [tex]\sigma[/tex], the sampling distribution of the sample means with size n can be approximated to a normal distribution with mean [tex]\mu[/tex] and standard deviation [tex]s = \frac{\sigma}{\sqrt{n}}[/tex].

For a skewed variable, the Central Limit Theorem can also be applied, as long as n is at least 30.

Mean (mu) that equals 100 with a standard deviation (sigma) of 18

[tex]\mu = 100, \sigma = 18[/tex]

Sample of 9:

This means that [tex]n = 9, s = \frac{18}{\sqrt{9}} = 6[/tex]

What will be the computed z-score with a sample mean (x-bar) of 106?

This is Z when X = 106. So

[tex]Z = \frac{X - \mu}{\sigma}[/tex]

By the Central Limit Theorem

[tex]Z = \frac{X - \mu}{s}[/tex]

[tex]Z = \frac{106 - 100}{6}[/tex]

[tex]Z = 1[/tex]

So Z = 1 is the answer.

hlo anyone free .... im bo r ed

d​

Answers

Step-by-step explanation:

Excuse me! Who r u? where r u frm? tell me tht frst.

Answer:

Oop

Step-by-step explanation:

I’m bored

can some0ne help me?

Answers

Answer:

(x - 2)/3

(x - 4)/-5 or (-x + 4)/5

Step-by-step explanation:

this is an inverse function, and to solve an inverse function you would :

swap x and g(x) without bringing the x coefficient with it, just simply swap the variables. Then, solve for g(x), and that's it

the first question's answer is :

g(x) = 3x + 2

x = 3(g(x)) + 2

x - 2 = 3(g(x))

(x - 2)/3 = g(x)

the second one is:

g(x) = 4 - 5x

x = 4 - 5(g(x))

x - 4 = -5(g(x))

(x-4)/-5 = g(x)

g(x) = 3x + 2

y = 3x + 2

x = 3y + 2

3y = x - 2

y = x/3 - 2/3

inverse g(x) = (x - 2) / 3

g(x) = 4 - 5x

y = 4 - 5x

x = 4 - 5y

5y = 4 - x

y = 4/5 - x/5

inverse g(x) = (4 - x) / 5

Solve this equation for x. Round your answer to the nearest hundredth.
1 = In(x + 7) ​

Answers

Answer:

[tex]\displaystyle x \approx -4.28[/tex]

General Formulas and Concepts:

Pre-Algebra

Equality Properties

Algebra II

Natural logarithms ln and Euler's number e

Step-by-step explanation:

Step 1: Define

Identify

[tex]\displaystyle 1 = ln(x + 7)[/tex]

Step 2: Solve for x

[Equality Property] e both sides:                                                                     [tex]\displaystyle e^1 = e^{ln(x + 7)}[/tex]Simplify:                                                                                                             [tex]\displaystyle x + 7 = e[/tex][Equality Property] Isolate x:                                                                            [tex]\displaystyle x = e - 7[/tex]Evaluate:                                                                                                            [tex]\displaystyle x = -4.28172[/tex]

e^1 = x+7

e - 7 = x

x = -4.28

please help me on this

Answers

Answer:

Median

Step-by-step explanation:

Using the median to measure central tendency, rather than the mean, is better for a skewed data set.

Since a skewed data set will have either very high or low extreme data points, the mean will be less representative and accurate when measuring central tendency.

Using the median will measure this better because it is not as vulnerable as the mean when there are extreme data points.

So, the answer is the median.

The answer is median his is because the mean value is depend on the correct media

The measure of angle tis 60 degrees.
What is the x-coordinate of the point where the
terminal side intersects the unit circle?
1
2
O
O
Isla Isla
2
DONE

Answers

Answer:

Step-by-step explanation:

Not a clear list of options and/or reference frame

Probably     0.5      if angle t is measured from the positive x axis.


In the figure above, AD and BE intersect at point C, and
the measures of angles B, D, and E are 98°, 81°, and 55°,
respectively. What is the measure, in degrees, of
angle A ? (Disregard the degree sign when gridding your
answer.)

Answers

answer in screenshot

what is sqrt 2x-3 = sqrt 3x-9

Answers

Answer:

x=6

Step-by-step explanation:

sqrt (2x-3) = sqrt (3x-9)

Square each side

(sqrt (2x-3))^2 = (sqrt (3x-9))^2

2x-3 = 3x-9

Subtract 2x from each side

2x-3-2x = 3x-2x-9

-3 = x-9

Add 9 to each side

-3+9 = x-9+9

6 =x

Check solution

sqrt (2*6-3) = sqrt (3*6-9)

sqrt (9) = sqrt (9)

3=3

Solution is valid

sqrt (2x-3) = sqrt (3x-9)

Square each side

(sqrt (2x-3))^2 = (sqrt (3x-9))^2

2x-3 = 3x-9

Subtract 2x from each side

2x-3-2x = 3x-2x-9

-3 = x-9

Add 9 to each side

-3+9 = x-9+9

6 =x

sqrt (2*6-3) = sqrt (3*6-9)

sqrt (9) = sqrt (9)

3=3

Therefore ans x = 6

Answered by Gauthmath must click thanks and mark brainliest

could someone please answer this? :) thank you

Answers

Answer:

The last one

Step-by-step explanation:

What we know:

We have a total of 16 coins

The 16 coins consist of dimes and quarters

The value of the coins is 3.10

The value of a dime is .10

The value of a quarter is .25

Using this information we can create a system of linear equations

First off we know that we have a total of 16 coins which consist of dimes and quarters

The number of Quarters can be represented by q and then number of dimes can be represented by d.

If we have a total of 16 coins then q + d must equal 16

So equation 1 is q + d = 16

Now we need to create a second equation

We know that the total value of the coins is 3.10 and we know that the coins consist of dimes and quarters

As you may know a quarter has a value of .25 cents and a dime has a value of 10 cents

If the total value of the coins is 3.10 the the number of dimes (d) times .10 + the number of Quarters times .25 must equal 3.10

This can also be written as

.25q + .10d = 3.10

So the two equations are

q + d = 16 and .25q + .10d = 3.10

These equations are shown in the last answer choice

Note: b is very similar to d

However the the value of the coins are incorrect in B

In B the value of the dime is represented by 10 which is not correct because the value of a dime is .10 not 10

A man starts repaying a loans with first insfallameny of rs.10 .If he increases the instalment by Rs 5 everything months, what amount will be paid by him in the 30the instalment.​

Answers

Answer:

30×5=150

so 150+10=160

thus his payment in the 30th installment is

rs.160

I need answering ASAP please

Answers

Answer:

The choose (D) 1/3

I hope I helped you^_^

Use absolute value to express the distance between -12 and -15 on the number line


A: |-12-(-15)|= -37
B: |-12-(-15)|= -3
C: |-12-(-15)|= 3
D: |-12-(-15)|= 27​

Answers

C
I-12-(-15)l
l-12+15l
l3l = 3

I'm interval notation please

Answers

9514 1404 393

Answer:

  (-2, 4]

Step-by-step explanation:

  -21 ≤ -6x +3 < 15 . . . . given

  -24 ≤ -6x < 12 . . . . . . subtract 3

  4 ≥ x > -2 . . . . . . . . . . divide by -6

In interval notation, the solution is (-2, 4].

__

Interval notation uses a square bracket to indicate the "or equal to" case--where the end point is included in the interval. A graph uses a solid dot for the same purpose. When the interval does not include the end point, a round bracket (parenthesis) or an open dot are used.

use quadratic formula to solve the following equation​

Answers

9514 1404 393

Answer:

  x = 2 or x = 9

Step-by-step explanation:

To use the quadratic formula, we first need the equation in standard form for a quadratic. We can get there by multiplying the equation by 3(x -3).

  2(3) +4(3(x -3)) = (x +4)(x -3)

  6 +12x -36 = x² +x -12

  x² -11x +18 = 0

Using the quadratic formula to find the solutions, we have ...

  [tex]x=\dfrac{-b\pm\sqrt{b^2-4ac}}{2a}=\dfrac{-(-11)\pm\sqrt{(-11)^2-4(1)(18)}}{2(1)}\\\\x=\dfrac{11\pm\sqrt{49}}{2}=\{2,9\}[/tex]

The solutions are x=2 and x=9.

A box with a square base and no top is to be made from a square piece of carboard by cutting 4 in. squares from each corner and folding up the sides. The box is to hold 1444 in3. How big a piece of cardboard is needed

Answers

Answer:

[tex]C=27inch\ by\ 27inch[/tex]

Step-by-step explanation:

Squares [tex]h=4inch[/tex]

Volume [tex]v=1444in^3[/tex]

Generally the equation for Volume of box is mathematically given by

 [tex]V=l^2h[/tex]

 [tex]1444=l^2*4[/tex]

 [tex]l^2=361[/tex]

 [tex]l=19in[/tex]

Since

Length of cardboard is

 [tex]l_c=19+4+4[/tex]

 [tex]l_c=27in[/tex]

Therefore

Dimensions of the piece of cardboard is

[tex]C=27inch\ by\ 27inch[/tex]

11. What is the midpoint of CD?
12. a. What are the exact lengths of
segments AB and CD?
b. How do the lengths of AB and CD
compare?
c. Is the following statement true or
false?
AB=CD

Answers

9514 1404 393

Answer:

  11. (-1.5, 3)

  12. √29, identical lengths, true they are congruent

Step-by-step explanation:

11. The midpoint is halfway between the end points. On a graph, you can count the grid squares between the ends of the segment and locate the point that is half that number from either end.

Points C and D differ by 2 in the y-direction, so the midpoint will be 1 unit vertically different from either C or D. That is, it will lie on the line y = 3. The segment CD intersects y=3 at x = -1.5, so the midpoint of CD is (-1.5, 3).

If you like, you can calculate the midpoint as the average of the end points:

  midpoint CD = (C +D)/2 = ((-4, 4) +(1, 2))/2 = (-3, 6)/2 = (-1.5, 3)

__

12. The exact length can be found using the Pythagorean theorem. The segment is the hypotenuse of a right triangle whose legs are the differences in x- and y-coordinates.

In the previous problem, we observed that the y-coordinates of C and D differed by 2. The x-coordinates differ by 5. Looking at segment AB, we see the same differences: x-coordinates differ by 5 and y-coordinates differ by 2. Then the lengths of each of these segments is ...

  AB = CD = √(2² +5²) = √29

a) The exact lengths of segments AB and CD are √29 units.

b) The lengths of the segments are identical

c) It is TRUE that the segments are congruent.

A school has 4 different after school activities planned in the fall Janet has time to participate in 2 of these activities. How many different pairs of after-school activities can Janet choose from the available activities?​

Answers

Answer:

6

Step-by-step explanation:

Of 4 options, Janet has to choose 2. This is combinations as A and B is the same as B and A.

Combinations formula gives us 4!/ 2!2! , or 6.

Amanufacturer of potato chips would like to know whether its bag filling machine works correctly at the 433 gram setting. It is believed that the machine is underfilling the bags. A 26 bag sample had a mean of 427 grams with a variance of 324. A level of significance of 0.05 will be used. Assume the population distribution is approximately normal. Is there sufficient evidence to support the claim that the bags are underfilled?

Answers

Answer:

There is not enough evidence to support the claim that the bags are under filled.

Step-by-step explanation:

Given :

Population mean, μ = 433

Sample size, n = 26

xbar = 427

Variance, s² = 324 ; Standard deviation, s = √324 = 18

The hypothesis :

H0 : μ = 433

H0 : μ < 433

The test statistic :

(xbar - μ) ÷ (s/√(n))

(427 - 433) / (18 / √26)

-6 / 3.5300904

T = -1.70

The Pvalue :

df = 26-1 = 25 ; α = 0.05

Pvalue = 0.0508

Since Pvalue > α ; WE fail to reject the Null and conclude that there is not enough evidence to support the claim that the bags are underfilled

Find the slope of the line containing the points (-3, 8) and (2, 4).

Answers

Answer:

-4/5

Step-by-step explanation:

The slope of the line is m=(4-8)/(2-(-3))=-4/5

*PLEASE HELP ME ILL GIVE BRAINLIST IF CORRECT*

Noah is playing a game where he must spin two wheels, each with 9 equal slices. There are 3 red slices, 3 green slices, 2 blue slices and 1 yellow slice on each wheel. If Noah spins and lands on a yellow slice on both wheels he wins, but if he lands on any other color, he loses. This information was used to create the following area model.


Is this a fair game? Why or why not?

A. Yes, the game is fair because Noah has equal probabilities of winning or losing.

B. Yes, the game is fair because Noah does not have equal probabilities of winning or losing.

C. No, the game is not fair because Noah has equal probabilities of winning or losing.

D. No, the game is not fair because Noah does not have equal probabilities of winning or losing.

Answers

Step-by-step explanation:

Yes, the game is fair because noah has equal probabilities of Winning

Answer:

No, the game is not fair because Noah does not have equal probabilities of winning or losing.

Step-by-step explanation:

Twice a number increased by the product of the number and fourteen results in forty eight

Answers

Answer:

Let x = the number. Then you have:

2x + 14x = 48 Collect like terms

16x = 48 Divide both sides by 16

x = 3

PLEASE MARK AS BRAINLIEST ANSWER

The number that satisfies the given statement is 3.

We are given that twice a number increased by the product of the number and 14 results in 48.

We will find the value of the number that we used in the given above statement.

Understand the meaning of the keywords used in the statement.

Increased means addition.

Product means multiplication.

Results mean equal to sign.

Let's write the given statement in equation form.

Consider P = the number

Twice a number = 2P

Increased =  +

Products of the number and 14 =  P x 14

Results in 48 = equals 48.

Combining all the above we get,

2P + P x 14 = 48

2P + 14P = 48

16P = 48

P = 48 / 16

P = 3

Thus the number that satisfies the given statement is 3.

Learn more about similar problems here:

https://brainly.com/question/17618748

#SPJ2

A ball thrown upwards hits a roof and returns back to the ground.

The upward movement is modeled by a function [tex]s=-t^2+3t+4[/tex]
s= −(t^2)+3t+4
and the downward movement is modeled by [tex]s=-t^2+3t+4[/tex]
s= −2(t^2)+t+7, where s is the distance (in metres) from the ground and t is the time in seconds.

Find the height of the roof from the ground.

Answers

Answer: 6 m

A ball thrown upwards from the altitude 4 m,

hits a roof and returns back to the ground.

upward movement:  s= −t²+3t+4

downward movement: s=-2t²+t+7

Step-by-step explanation:

Let's calculate the intersection:

[tex]- t^2+3t+4 =-2t^2+t+7\\\\t^2+2t-3=0\\\\t^2+3t-t-3=0\\\\t(t+3)-(t+3)=0\\\\(t+3)(t-1)=0\\\\t=-3 \ (exclude)\ or\ t=1\\\\if\ t=1 \ then\ s=-1^2+3*1+4=6\\\\height\ is\ 6\ m.\\[/tex]

Sorry, i have forgotten the picture.

Other Questions
A sofa regularly sells for $760. The sale price is $676.40. Find the percent decrease of the sale price from the regular price. What does it mean for a human to express their genes? What is the least common denominator that will allow you to combine the constant terms? 10 21 35 or 42 HELP PLEASEEE!!!! ASAP!!!!! 7. If the supplement of an angle is 69, then find its complement. Find the length of the arc round your answer to the nearest 10th NOW ASAP PLEASE NEED FAST ANSWERRRRRRRRRRRRRRRR What is figurative language?A. MetaphorB. Imaginative writingC. Flowery language which has realistic descriptionsD. Subtle language that cannot be taken literally a rice cooker was sold for $60 after a discount of 60% waht was the usual price of the rice cooker Which of the following is the number of sides for a regular polygon that willnot form a regular tessellation?A. 6B. 4C. 7D. 3 What is the second integral of [tex] \frac{1}{x} [/tex]? arrange the resistance r1 r2 and r3 in increasing order. 5. Find the area of the shaded part in this figure. A state of economic scarcity exists when consumers: A. do not have enough resources to satisfy all of their wants.B. save their money in banks instead of spending it. C. borrow too much money to buy unnecessary products. D. have their economic freedoms restricted by the government. The following refers to units processed by a breakfast cereal maker in August. Compute the total equivalent units of production with respect to conversion for August using the weighted-average inventory method. Tons of Product Percent of Conversion Added Beginning Work in Process 230,000 60%Goods started 570,000 100%Goods completed 620,000 100Ending Work in Process 180,000 70%a. 758,000b. 800,000c. 620,000d. 746,000 Cox Engineering performs cement core tests in its laboratory. The following standards have been set for each core test performed:Standard Hours or Quantity or Rate Standard PriceDirect materials 3 pounds $0.75 per poundDirect labor 0.4 hours $12 per hourVariable manufacturing overhead 0.4 hours $9 per hourDuring March, the laboratory performed 2,000 core tests. On March 1 no direct materials (sand) were on hand. Variable manufacturing overhead is assigned to core tests on the basis of standard direct labor-hours.The following events occurred during March:[1] 8,600 pounds of sand were purchased at a cost of $7,310.[2] 7,200 pounds of sand were used for core tests.[3] 840 actual direct labor-hours were worked at a cost of $8,610.[4] Actual variable manufacturing overhead incurred was $3,200.REQUIRED: (NOTE: You must show your computations)(a) The materials price variance for March is:(b) The materials quantity variance for March is:(c) The labor rate variance for March is:(d) The labor efficiency variance for March is:(e) The variable overhead efficiency variance for March is: Omg who can help me? Its due tomorrow Based on how they each acted to the trials,witch substance (s)would most likely be metallic Which area appears only on the posterior part of the body? a)scapular b)patellar c)umbilical d)sternal D. He no longer thinks about her.